Coeficientes de expansión de un estado arbitrario en el espacio de Hilbert de estados de una partícula

Estaba revisando mis notas sobre las representaciones unitarias irreducibles del grupo de Poincaré y la posterior construcción de estados de una partícula y me topé con los siguientes pasos en el método de representaciones inducidas: ( H es el espacio de Hilbert y H pag es el subespacio de estados de una partícula con momentos p. S k es solo el pequeño grupo del impulso estándar k . L ( pag ) es el impulso estándar de Lorentz; L ( pag ) k = pag )

(1) Deja | k , σ ser una base para H k y deja D sea ​​el irrep unitario (finito) de S k inducida por la irrep unitaria tu . Entonces D actúa sobre H k mezclando los estados de polarización de espín,

tu ( h ) | k , σ = σ D σ σ ( h ) | k , σ ,       h S k ,       tu ( h ) tu ( h , 0 ) .
(2) Definir la base para H pag por
| pag , σ = tu ( L ( pag ) ) | k , σ .
(3) La acción de las transformaciones de Lorentz sobre estos estados es entonces
tu ( Λ ) | pag , σ = σ D σ σ ( h ( Λ , pag ) ) | Λ pag , σ
dónde h ( Λ , pag ) = L 1 ( Λ pag ) Λ L ( pag ) S k se conoce como la rotación de Wigner.

(4) Finalmente, extendemos esta acción a un estado arbitrario en el espacio de Hilbert | Ψ = pag , σ Ψ σ ( pag ) | pag , σ H para algunos coeficientes Ψ σ ( pag ) por,

tu ( Λ ) | Ψ = pag , σ [ tu ( Λ ) Ψ ] σ ( pag ) | pag , σ ,  dónde  [ tu ( Λ ) Ψ ] σ ( pag ) = σ D σ σ ( h ( Λ , Λ 1 pag ) ) Ψ σ ( Λ 1 pag ) .

Estas son mis propias notas que están influenciadas en gran medida por la construcción de Weinberg, por lo que pueden ser incorrectas. Entiendo todos los pasos excepto una parte del paso 4; La parte donde digo | Ψ = pag , σ Ψ σ ( pag ) | pag , σ H para algunos coeficientes Ψ σ ( pag ) . ¿Cuáles son estos coeficientes? ¿Son las funciones de onda? ¿Dependen continuamente de pag , si es así, la suma debe superar pag ser reemplazada por una integral? En caso de que estos coeficientes sean integrables al cuadrado; Ψ σ ( pag ) L 2 ( Σ metro , m , C ) dónde Σ metro es la capa de masa y m una medida invariante de Lorentz? ¿Es correcta esta expansión de un estado arbitrario?

Respuestas (1)

Por construcción, el | pag , σ son una base de su espacio vectorial. Entonces todos sus estados son combinaciones lineales de estos vectores base. Sin embargo, tienes razón al preguntar sobre la naturaleza de los coeficientes ya que no es tan simple en este caso:

Hay tres nociones diferentes de "base" para un espacio de Hilbert mecánico cuántico, que desafortunadamente pocos textos de física se molestan en distinguir correctamente:

  1. Una base de dimensión finita. En caso de que su espacio vectorial sea de dimensión finita, tiene un número finito de vectores base | v i , i = 1 , , norte y todo vector es una combinación lineal

    | v = i = 1 norte C i | v i , C i C .
    Esta definición no tiene más problemas.

  2. Una base de Hilbert. En caso de que su espacio vectorial sea de dimensión infinita y un espacio de Hilbert separable, puede tomar una cantidad contable de vectores base | v i , i norte y todo vector es una combinación lineal infinita

    | v = i = 1 C i | v i , C 2 ( C ) ,
    dónde C 2 ( C ) significa que la secuencia C = ( C i ) i norte es sumable al cuadrado como i = 1 | C i | 2 < .

  3. Una base amañada 1 . De nuevo, en el caso de que el espacio vectorial sea de dimensión infinita, tenemos una serie de "vectores" "base" | v ( X ) , X R tal que cada vector es una "combinación lineal" "incontablemente infinita"

    " | v = ψ ( X ) | v ( X ) d X " ,
    dónde ψ ( X ) L 2 ( R ) es una función compleja de integración cuadrada, es decir, una "función de onda". La razón de todas las comillas de miedo aquí es que esta noción es "fácil" de escribir para el físico, pero muy difícil de entender para el matemático. Desde v ( X ) | v ( X ) = d ( X X ) , su producto interno está mal definido y estas cosas no son vectores en el espacio de Hilbert propiamente dicho, sino que se encuentran fuera de él en la mayor parte de un espacio de Hilbert amañado , vea también esta pregunta y respuesta . La integral también es difícil de hacer rigurosa, por lo que también podría reemplazarla por una suma como lo hace Weinberg, ciertamente no es ni una integral ni una suma como las entenderíamos ingenuamente, pero es claramente una generalización de la suma como la integral. Sin embargo, debe estar familiarizado con esto de la mecánica cuántica básica, donde generalmente no tenemos reparos en escribir 1 = | X X | d X , y también ψ ( X ) = X | ψ y | ψ = X | ψ | X d X .

En este caso, su | pag , σ son una mezcla de una base finita y una base amañada: solo hay un número finito de opciones para giros de una partícula σ , pero hay innumerables opciones de impulso. Entonces, para fijo pag tenemos Φ ( pag ) C norte para norte = 2 s + 1 para s el espín total de la partícula. para fijo σ , tenemos Φ σ L 2 ( Σ metro ) . Entonces, en conjunto, Φ σ ( pag ) son los valores de un C norte -función valorada que es integrable al cuadrado en cada componente, y a la que solo puede alimentar momentos pag con el correcto pag 2 = metro 2 relación.


1 Esta terminología es mi invención personal. Si hay un nombre propio para estas cosas, estaría feliz de escucharlo.